Find value of a and b such that F(x) is a valid cumulative distribution function

Click For Summary
To determine the values of a and b for F(x) to be a valid cumulative distribution function, it is essential that F(x) approaches 1 as x approaches infinity and 0 as x approaches negative infinity. The function must also be right continuous and non-negative. Continuity at x=0 can be checked by evaluating the limits of both forms of F as x approaches 0. While some restrictions on a and b can be identified, the exact calculations for these constants remain unclear. Further clarification on the continuity requirement may help in solving for a and b.
chessmath
Messages
20
Reaction score
0
Hi
I have a question , the question asks find value of a and b such that F(x) is a valid cumulative distribution function?

1-a*exp(-x/b) x≥0
F(x)=
a*exp(x/b) x<0

My attempt to solve the problem:

I know F(x) when x goes to ∞ in 1 and when x goes to -∞ is 0. also I know F(x) should be right continuous and it is non-negative. However, non of them help me to find even one of the constants, any help will be appreciated ?
 
Last edited by a moderator:
Physics news on Phys.org


chessmath said:
Hi
I have a question , the question asks find value of a and b such that F(x) is a valid cumulative distribution function?

1-a*exp(-x/b) x≥0
F(x)=
a*exp(x/b) x<0

My attempt to solve the problem:

I know F(x) when x goes to ∞ in 1 and when x goes to -∞ is 0. also I know F(x) should be right continuous and it is non-negative. However, non of them help me to find even one of the constants, any help will be appreciated ?

If you want it to be continuous at x=0 what are the limits of both forms of F as x->0? Though actually if you only have right continuous, I don't think you have that restriction. Hmm. Not sure. I can see some restrictions on the values of a and b, but not how to calculate them.
 
Last edited:
Question: A clock's minute hand has length 4 and its hour hand has length 3. What is the distance between the tips at the moment when it is increasing most rapidly?(Putnam Exam Question) Answer: Making assumption that both the hands moves at constant angular velocities, the answer is ## \sqrt{7} .## But don't you think this assumption is somewhat doubtful and wrong?

Similar threads

  • · Replies 11 ·
Replies
11
Views
2K
Replies
2
Views
1K
  • · Replies 11 ·
Replies
11
Views
2K
  • · Replies 20 ·
Replies
20
Views
3K
  • · Replies 5 ·
Replies
5
Views
2K
  • · Replies 8 ·
Replies
8
Views
1K
Replies
6
Views
2K
  • · Replies 13 ·
Replies
13
Views
13K
  • · Replies 3 ·
Replies
3
Views
1K
  • · Replies 1 ·
Replies
1
Views
1K